Finał 2009 Rozwiązania

<br />
\centerline{POWSZECHNY KONKURS INTERNETOWY dla uczniów szkół<br />
średnich - Matematyka} \centerline{Finał X edycji - 25 kwietnia 2009}<br />
\centerline{Przykładowe rozwiązania}<br />





\textbf{\underline{Zadanie 1.}}<br />
Znaleźć wszystkie pary $(x,y)$ liczb rzeczywistych spełniające układ równań<br />
$$\left\{\begin{array}{l} \displaystyle \sqrt{\frac{x}{y}}+\sqrt{\frac{y}{x}}=\frac{7}{\sqrt{xy}}+1 \\ \\ x\sqrt{xy}+y\sqrt{xy}=78 \end{array}\right.$$<br />
\\ \\<br />
\textbf{\underline{Rozwiązanie}}<br />
\\ \\ Z pierwszego równania otrzymujemy, że $x,y$ muszą być tego samego znaku (i nie mogą być zerami). Jednak gdy rozważymy drugie równanie i przyjmiemy, że $x,y$ są ujemne, wtedy lewa strona równania będzie ujemna, a prawa dodatnia - sprzeczność.<br />
\\ Stąd wnioskujemy, że $x,y$ muszą być liczbami dodatnimi.<br />
\\ Pomnóżmy pierwsze równanie przez $\sqrt{xy}$:<br />
$$\left\{\begin{array}{l}<br />
x+y=7+\sqrt{xy} \\ \sqrt{xy}(x+y)=78 \end{array}\right.$$<br />
\\ Podstawmy $u=x+y$, $v=\sqrt{xy}$ (oczywiście $u,v>0$):<br />
$$\left\{\begin{array}{l}<br />
u=7+v \\ uv=78 \end{array}\right.$$<br />
$$(7+v)v=78$$<br />
$$v^2+7v-78=0$$<br />
$$(v-6)(v+13)=0$$<br />
$$v=6\quad\vee\quad v=-13$$<br />
\\ Ale $v>0$:<br />
$$\left\{\begin{array}{l}<br />
v=6 \\<br />
u=13 \end{array}\right.$$<br />
$$\left\{\begin{array}{l}<br />
\sqrt{xy}=6 \\<br />
x+y=13 \end{array}\right.$$<br />
$$\left\{\begin{array}{l}<br />
xy=36 \\<br />
y=13-x \end{array}\right.$$<br />
$$x(13-x)=36$$<br />
$$x^2-13x+36=0$$<br />
$$(x-4)(x-9)=0$$<br />
\\ Dla $x=4$ mamy $y=9$<br />
\\ Dla $x=9$ mamy $y=4$<br />
\\ Rozwiązaniami są dwie pary: $(x,y)=(4,9)$, $(x,y)=(9,4)$.<br />





\textbf{\underline{Zadanie 2.}}<br />
W trójkącie równoramiennym wysokości względem podstawy i ramienia mają długości $12$ cm i odpowiednio $14,4$ cm. Obliczyć stosunek promienia koła wpisanego w ten trójkąt do promienia koła na nim opisanego.<br />
\\ \\<br />
\textbf{\underline{Rozwiązanie}}<br />
\\ \\ Przyjmijmy oznaczenia jak na rysunku:<br />
\\<br />

</p>
<p> Ponadto niech $|AB|=a$, $|BC|=b$, wtedy $|DB|=\frac{a}{2}$.<br />
\\ Z tw. Pitagorasa w $\Delta DBC$:<br />
$$\left(\frac{a}{2}\right)^2+12^2=b^2$$<br />
$$b=\sqrt{144+\frac{a^2}{4}}$$<br />
\\ Ponieważ $\Delta ABE\sim\Delta CDB$ (k-k-k), to:<br />
$$\frac{|AE|}{|AB|}=\frac{|CD|}{|CB|}$$<br />
$$\frac{14.4}{a}=\frac{12}{\sqrt{144+\frac{a^2}{4}}}$$<br />
\\ Stąd otrzymamy $a=18$, więc $b=15$.<br />
\\ Wtedy:<br />
$$P_{ABC}=\frac{1}{2}a\cdot 12=108$$<br />
\\ Korzystając ze wzorów:<br />
$$P_{ABC}=\frac{(a+b+b)r}{2}\qquad P_{ABC}=\frac{abb}{4R}$$<br />
\\ Gdzie $r,R$ to odpowiednio długości promieni okręgów wpisanego i opisanego:<br />
$$108=\frac{48r}{2}\qquad 108=\frac{18\cdot 15\cdot 15}{4R}$$<br />
$$r=\frac{9}{2}\qquad R=\frac{75}{8}$$<br />
\\ Więc szukany stosunek wynosi:<br />
$$\frac{r}{R}=\frac{12}{25}$$<br />





\textbf{\underline{Zadanie 3.}}<br />
W talii złożonej z $52$ kart jest po $13$ pików, kierów, kar i trefli. W każdym kolorze jest as, król, dama, walet i karty od dziesiątki do dwójki. W grze w pokera fulem nazywamy układ $5$ kart składający się z trzech kart tego samego typu oraz pary kart tego samego typu, np.: $3$ króle i $2$ asy. Wylosowanie dowolnych $5$ kart z tej talii jest tak samo prawdopodobne. Jakie jest prawdopodobieństwo wylosowania fula?<br />
\\ \\<br />
\textbf{\underline{Rozwiązanie}}<br />
\\ \\ Niech $\Omega$ - oznacza wszystkie możliwości wylosowania pięciu kart z talii.<br />
\\ Oczywiście:<br />
$$|\Omega|={52\choose 5}$$<br />
\\ Niech $A$ - oznacza wszystkie możliwości uzyskania fula.<br />
\\ Wtedy $\displaystyle |A|={13\choose 1}{4\choose 3}{12\choose 1}{4\choose 2}$.<br />
\\ Najpierw wybieramy jedną z trzynastu figur (od $2$ do asa), następnie z czterech kolorów dobieramy trzy, w ten sposób otrzymamy trzy karty, które utworzą nam część fula. Następnie wybieramy jedną z dwunastu pozostałych figur, a następnie z czterech kolorów dobieramy dwa.<br />
\\<br />
\\ Stąd szukane prawdopodobieństwo jest równe:<br />
$$P(A)=\frac{|A|}{|\Omega|}=\ldots=\frac{4}{4165}$$<br />





\textbf{\underline{Zadanie 4.}}<br />
Wykazać, że liczba<br />
$$3\cdot 7\cdot 11\cdot 29\cdot 40\cdot 299\cdot\sin 10^{\circ}\cdot \cos 160^{\circ}\cdot \sin 130^{\circ}\cdot\left[\frac{1}{2}\log_{\frac{1}{2}}\left(2+\sqrt{3}\right)+\log_{\frac{1}{2}}\left(\sqrt{6}-\sqrt{2}\right)\right]$$<br />
jest liczbą naturalną. Ile dzielników będących liczbami naturalnymi ma ta liczba?<br />
\\ \\<br />
\textbf{\underline{Rozwiązanie}}<br />
\\ \\ Obliczmy (będziemy korzystać ze wzoru $\sin\alpha\cos\alpha=\frac{1}{2}\sin 2\alpha$<br />
oraz ze wzorów redukcyjnych):<br />
$$\sin 10^{\circ}\cos 160^{\circ}\sin 130^{\circ}=<br />
\sin 10^{\circ}\cos (180^{\circ}-160^{\circ})\sin (90^{\circ}+40^{\circ})=$$ $$=<br />
-\sin 10^{\circ}\cos 20^{\circ}\cos 40^{\circ}=<br />
-\frac{\cos 10^{\circ}\sin 10^{\circ}\cos 20^{\circ}\cos 40^{\circ}}{\cos 10^{\circ}}=$$ $$=<br />
-\frac{\frac{1}{2}\sin 20^{\circ}\cos 20^{\circ}\cos 40^{\circ}}{\cos 10^{\circ}}=<br />
-\frac{\frac{1}{4}\sin 40^{\circ}\cos 40^{\circ}}{\cos ^{\circ}}=$$ $$=<br />
-\frac{\frac{1}{8}\sin 80^{\circ}}{\cos 10^{\circ}}=<br />
-\frac{\frac{1}{8}\cos 10^{\circ}}{\cos 10^{\circ}}=<br />
-\frac{1}{8}$$<br />
\\ Ponadto:<br />
$$\frac{1}{2}\log_{\frac{1}{2}}\left(2+\sqrt{3}\right)+\frac{1}{2}\log_{\frac{1}{2}}\left(\sqrt{6}-\sqrt{2}\right)^2=<br />
\frac{1}{2}\left(\log_{\frac{1}{2}}\left(2+\sqrt{3}\right)+\log_{\frac{1}{2}}\left(6-2\sqrt{12}+2\right)\right)=$$ $$=<br />
\frac{1}{2}\left(\log_{\frac{1}{2}}(2+\sqrt{3})+\log_{\frac{1}{2}}(8-4\sqrt{3})\right)=<br />
\frac{1}{2}\log_{\frac{1}{2}}\left[(2+\sqrt{3})4(2-\sqrt{3})\right]=$$ $$=<br />
\frac{1}{2}\log_{\frac{1}{2}}4=<br />
\log_{\frac{1}{2}}2=-1$$<br />
\\ Nasza liczba jest więc równa:<br />
$$M=3\cdot 7\cdot 11\cdot 29\cdot 8\cdot 5\cdot 13\cdot 23\cdot\left(-\frac{1}{8}\right)\cdot (-1)=<br />
3\cdot 7\cdot 11\cdot 29\cdot 5\cdot 13\cdot 23$$<br />
\\ Jako iloczyn liczb naturalnych liczba $M$ jest liczbą naturalną.<br />
\\ Liczbę $M$ przedstawiliśmy jako iloczyn siedmiu różnych liczb pierwszych.<br />
\\ By utworzyć dzielnik liczby $M$ musimy wybrać jakiś podzbiór z tych $7$ liczb pierwszych i wymnażając liczby z tego podzbioru utworzyć dzielnik (gdy wybierzemy zbiór pusty, przymijmy, że dzielnikiem jest $1$). Stąd ilość dzielników liczby $M$<br />
jest równa ilości podzbiór zbioru $7$-elementowego, czyli $2^7=128$.<br />





\textbf{\underline{Zadanie 5.}}<br />
Dany jest czworościan $ABCD$ o krawędziach długości: $|BC|=a$, $|AC|=b$, $|AB|=c$, $|AD|=d$, $|BD|=e$, $|CD|=f$. Punkt $S$ jest środkiem ciężkości trójkąta $ABC$.<br />
\\ Dowieść, że<br />
$$|DS|=\frac{1}{3}\sqrt{3d^2+3e^2+3f^2-a^2-b^2-c^2}$$<br />
\\ \\<br />
\textbf{\underline{Rozwiązanie}}<br />
\\ \\ Przyjmijmy oznaczenia jak na rysunku:<br />

 Skoro $S$ jest środkiem cięzkości trójkąta $ABC$, to jest to punkt przecięcia się środkowych w tym trójkącie.<br />
\begin{itemize}<br />
\item Popatrzmy na przekrój $BCD$, oczywiście $|BE|=\frac{a}{2}$.<br />
\\ Z tw. cosinusów w trójkątach: $BCD$ oraz $BED$ otrzymujemy układ równań:<br />
$$\left\{\begin{array}{l}<br />
|DE|^2=\frac{a^2}{4}+e^2-2\cdot\frac{a}{2}\cdot e\cos\alpha \\<br />
f^2=a^2+e^2-2ae\cos\alpha \end{array}\right.$$<br />
\\ Mnożymy pierwsze równanie przez $2$:<br />
$$\left\{\begin{array}{l}<br />
2|DE|^2=\frac{a^2}{2}+2e^2-2ae\cos\alpha \\<br />
f^2=a^2+e^2-2ae\cos\alpha \end{array}\right.$$<br />
\\ Odejmujemy stronami:<br />
$$2|DE|^2-f^2=-\frac{a^2}{2}+e^2$$<br />
$$|DE|^2=\frac{2e^2+2f^2-a^2}{4}$$<br />
\item Popatrzmy na przekrój $BCA$, w analogiczny sposób wyznaczamy długość $|AE|$:<br />
$$|AE|^2=\frac{2b^2+2c^2-a^2}{4}$$<br />
\item Popatrzmy na przekrój $AED$. Oczywiście $|AS|=\frac{2}{3}|AE|$ (środkowe przecinają się w jednym punkcie, który dzieli je w stosunku $2:1$, licząc od wierzchołka w trójkącie).<br />
\\ Analogicznie jak wcześniej stosujemy tw. cosinusów dla trójkątów $ASD$ oraz $AED$:<br />
$$\left\{\begin{array}{l}<br />
|DS|^2=d^2+\left(\frac{2}{3}\right)^2|AE|^2-2d\cdot\frac{2}{3}|AE|\cos\gamma \\<br />
|DE|^2=d^2+|AE|^2-2d|AE|\cos\gamma \end{array}\right.$$<br />
\\ Mnożymy pierwsze równanie przez $\frac{3}{2}$ i odejmujemy stronami równania:<br />
$$\frac{3}{2}|DS|^2-|DE|^2=\frac{1}{2}d^2-\frac{1}{3}|AE|^2$$<br />
\\ Wykorzystując wcześniej obliczone wielkości $|DE|^2$ oraz $|AE|^2$ otrzymamy:<br />
$$|DS|^2=\frac{1}{9}(3d^2+3e^2+3f^2-a^2-b^2-c^2)$$<br />
$$|DS|=\frac{1}{3}\sqrt{3d^2+3e^2+3f^2-a^2-b^2-c^2}$$<br />

<br />
\centerline{\bf Za rozwiązanie każdego zadania można uzyskać<br />
maksymalnie 20 punktów}<br />